Difference between revisions of "2017 AMC 10B Problems/Problem 13"

m (Solution)
(Solution)
Line 6: Line 6:
 
==Solution==
 
==Solution==
 
By PIE (Property of Inclusion/Exclusion), the answer is <math>10+13+9-9-20= \boxed{\textbf{(C) } 3}</math>.
 
By PIE (Property of Inclusion/Exclusion), the answer is <math>10+13+9-9-20= \boxed{\textbf{(C) } 3}</math>.
 +
 +
<math>|A_1 \cup A_2 \cup A_3| = \sum |A_i| - \sum |A_i \cap A_j| + |A_1 \cap A_2 \cap A_3|.</math>
 +
Number of ppl in at least two sets is <math>\sum |A_i \cap A_j| - 2|A_1 \cap A_2 \cap A_3| = 9</math>
 +
So, <math>20 = (10 + 13 + 9) - (9 + 2x) + x,</math> which gives <math>x = 3.</math>
  
 
==See Also==
 
==See Also==
 
{{AMC10 box|year=2017|ab=B|num-b=12|num-a=14}}
 
{{AMC10 box|year=2017|ab=B|num-b=12|num-a=14}}
 
{{MAA Notice}}
 
{{MAA Notice}}

Revision as of 23:15, 28 November 2019

Problem

There are $20$ students participating in an after-school program offering classes in yoga, bridge, and painting. Each student must take at least one of these three classes, but may take two or all three. There are $10$ students taking yoga, $13$ taking bridge, and $9$ taking painting. There are $9$ students taking at least two classes. How many students are taking all three classes?

$\textbf{(A)}\ 1\qquad\textbf{(B)}\ 2\qquad\textbf{(C)}\ 3\qquad\textbf{(D)}\ 4\qquad\textbf{(E)}\ 5$

Solution

By PIE (Property of Inclusion/Exclusion), the answer is $10+13+9-9-20= \boxed{\textbf{(C) } 3}$.

$|A_1 \cup A_2 \cup A_3| = \sum |A_i| - \sum |A_i \cap A_j| + |A_1 \cap A_2 \cap A_3|.$ Number of ppl in at least two sets is $\sum |A_i \cap A_j| - 2|A_1 \cap A_2 \cap A_3| = 9$ So, $20 = (10 + 13 + 9) - (9 + 2x) + x,$ which gives $x = 3.$

See Also

2017 AMC 10B (ProblemsAnswer KeyResources)
Preceded by
Problem 12
Followed by
Problem 14
1 2 3 4 5 6 7 8 9 10 11 12 13 14 15 16 17 18 19 20 21 22 23 24 25
All AMC 10 Problems and Solutions

The problems on this page are copyrighted by the Mathematical Association of America's American Mathematics Competitions. AMC logo.png